Medical Surgery Test + i love you po sorry ulit hehe!

Pataasin ang iyong marka sa homework at exams ngayon gamit ang Quizwiz!

Which of the following causes is the most common cause for sudden cardiac death? a. Ventricular tachycardia b. Aortic stenosis c. Hypertrophic cardiomyopathy d. Angina

ANS: A Acute ventricular dysrhythmias (e.g., ventricular tachycardia, ventricular fibrillation) cause the majority of cases of SCD. Less commonly, SCD occurs because of a primary left ventricular outflow obstruction (e.g., aortic stenosis, hypertrophic cardiomyopathy) or extreme slowing of the heart (bradycardia).

The health care provider recommends a carotid endarterectomy for a client with carotid atherosclerosis and a history of transient ischemic attacks (TIAs). The client asks the nurse to describe the procedure. Which of the following responses by the nurse is appropriate? a. "The carotid endarterectomy involves surgical removal of plaque from an artery in the neck." b. "The diseased portion of the artery in the brain is removed and replaced with a synthetic graft." c. "A wire is threaded through an artery in the leg to the clots in the carotid artery and the clots are removed." d. "A catheter with a deflated balloon is positioned at the narrow area, and the balloon is inflated to flatten the plaque."

ANS: A In a carotid endarterectomy, the carotid artery is incised and the plaque is removed. The response beginning, "The diseased portion of the artery in the brain is removed" describes an arterial graft procedure. The answer beginning, "A catheter with a deflated balloon is positioned at the narrow area" describes an angioplasty. The final response beginning, "A wire is threaded through the artery" describes the MERCI procedure.

The nurse is caring for a client who has recently started taking rosuvastatin and niacin who reports all of these symptoms to the nurse. Which of the following finding is most important to communicate to the health care provider? a. Generalized muscle aches and weakness b. Skin flushing after taking the medications c. Dizziness when changing positions quickly d. Nausea when taking the drugs before eating

ANS: A Muscle aches and weakness may indicate myopathy and rhabdomyolysis, which have caused acute renal failure and death in some clients who have taken the statin medications. These symptoms indicate that the rosuvastatin may need to be discontinued. The other symptoms are common adverse effects when taking niacin, and although the nurse should follow up with the client, they do not indicate that a change in medication is needed.

During a visit to a client with chronic heart failure, the home care nurse finds that the client has ankle edema, a 2 kg weight gain, and complains of "feeling too tired to do anything." Based on these data, which of the following is the best nursing diagnosis for the client? a. Activity intolerance related to physical deconditioning b. Disturbed body image related to alteration in self-perception c. Impaired skin integrity related to alteration in fluid volume (peripheral edema) d. Ineffective breathing pattern related to respiratory muscle fatigue

ANS: A The client's statement supports the diagnosis of activity intolerance. There are no data to support the other diagnoses, although the nurse will need to assess for other client problems.

. Following an acute myocardial infarction, a previously healthy client develops clinical manifestations of heart failure. The nurse anticipates discharge teaching will include information about which of the following medications? a. Angiotensin-converting enzyme (ACE) inhibitors b. Digitalis preparations c. b-adrenergic agonists d. Calcium channel blockers

ANS: A ACE inhibitor therapy is currently recommended to prevent the development of heart failure in clients who have had a myocardial infarction and as a first-line therapy for clients with chronic heart failure. Digoxin therapy for heart failure is no longer considered a first-line measure. Calcium channel blockers are not generally used in the treatment of heart failure. The b-adrenergic agonists such as dobutamine are administered through the IV route and are not used as initial therapy for heart failure.

The nurse is planning expected outcomes for a client with thromboangiitis obliterans (Buerger's disease). Which of the following outcomes has the highest priority for this client? a. Cessation of smoking b. Control of serum lipid levels c. Maintenance of appropriate weight d. Demonstration of meticulous foot care

ANS: A Absolute cessation of nicotine use is needed to reduce the risk for amputation in clients with Buerger's disease. Other therapies have limited success in treatment of this disease

6. The nurse is caring for a client who is experiencing a hypertensive crisis and is receiving sodium nitroprusside. Which of the following time frequencies should the nurse assess the clients' blood pressure and pulse during the initial administration of this medication? a. 2-3 minutes b. 5-10 minutes c. 15-30 minutes d. Hourly

ANS: A Administered intravenously, the drugs have a rapid (within seconds to minutes) onset of action. The client's BP and pulse should be taken every 2-3 minutes during the initial administration of these drugs.

The nurse is providing health-promotion teaching related to heart health and is explaining modifiable and nonmodifiable risk factors. Smoking is a modifiable risk factor. After a diagnosis of cardiovascular-related illness, approximately what percentage of clients that were smokers quit? a. 5 b. 15 c. 25 d. 50

ANS: A After diagnosis of a cardiovascular-related illness, fewer than 5% of Canadians quit smoking.

The nurse is caring for a client who has recently had a stroke. When reviewing the clients' laboratory report, which of the following results should the nurse report the health care provider? a. PaCO2 51 mm Hg b. pH 7.41 c. PaO2 96 mm Hg d. WBC 9.2 ´ 109 /L

ANS: A All of the lab values are within normal range except for the PaCO2. A high PaCO2 is a potent vasodilator that increases cerebral blood flow.

A client is admitted to the hospital with a diagnosis of chronic venous insufficiency. Which of the following client statements is most consistent with this diagnosis? a. "I can't get my shoes on at the end of the day." b. "I can never seem to get my feet warm enough." c. "I wake up during the night because my legs hurt." d. "I have burning leg pains after I walk three blocks."

ANS: A Because the edema associated with venous insufficiency increases when the client has been standing, shoes will feel tighter at the end of the day. The other client statements are characteristic of peripheral artery disease (PAD).

A client with heart failure has a new order for captopril 12.5 mg PO. After administering the first dose and teaching the client about captopril, which statement by the client indicates that teaching has been effective? a. "I will call for help when I need to get up to use the bathroom." b. "I will be sure to take the medication after eating something." c. "I will need to include more high-potassium foods in my diet." d. "I will expect to feel more short of breath for the next few days."

ANS: A Captopril can cause hypotension, especially after the initial dose, so it is important that the client not get up out of bed without assistance until the nurse has had a chance to evaluate the effect of the first dose. The ACE inhibitors are potassium sparing, and the nurse should not teach the client to increase sources of dietary potassium. Increased shortness of breath is expected with initiation of b-blocker therapy for heart failure, not for ACE inhibitor therapy. ACE inhibitors are best absorbed when taken an hour before eating.

. The nurse has initiated discharge teaching for a client who is to be maintained on warfarin following hospitalization for venous thrombo-embolism (VTE). Which of the following client statements indicate that additional teaching is required? a. "I should reduce the amount of green, leafy vegetables that I eat." b. "I should wear a Medic Alert bracelet stating that I take warfarin." c. "I will need to have blood tests routinely to monitor the effects of the warfarin." d. "I will check with my health care provider before I begin or stop any medication."

ANS: A Clients taking warfarin are taught to follow a consistent diet with regard to foods that are high in vitamin K, such as green, leafy vegetables. The other client statements are accurate.

The nurse is providing teaching to a client about the use of atenolol in preventing anginal episodes. Which of the following client statements indicate that the teaching has been effective? a. "It is important not to suddenly stop taking the atenolol." b. "Atenolol will increase the strength of my heart muscle." c. "I can expect to feel short of breath when taking atenolol." d. "Atenolol will improve the blood flow to my coronary arteries."

ANS: A Clients who have been taking b-blockers can develop intense and frequent angina if the medication is suddenly discontinued. Atenolol decreases myocardial contractility. Shortness of breath that occurs when taking b-blockers for angina may be due to bronchospasm and should be reported to the health care provider. Atenolol works by decreasing myocardial oxygen demand, not by increasing blood flow to the coronary arteries.

Which of the following information about a client who has been receiving fibrinolytic therapy for an acute myocardial infarction is most important for the nurse to communicate to the health care provider? a. No change in the client's chest pain b. A large bruise at the client's IV insertion site c. A decrease in ST segment elevation on the electrocardiogram (ECG) d. An increase in cardiac enzyme levels since admission

ANS: A Continued chest pain suggests that the fibrinolytic therapy is not effective and that other interventions such as percutaneous coronary intervention (PCI) may be needed. Bruising is a possible adverse effect of fibrinolytic therapy, but it is not an indication that therapy should be discontinued. The decrease of the ST segment elevation indicates that fibrinolysis is occurring and perfusion is returning to the injured myocardium. An increase in cardiac enzyme levels is expected with reperfusion and is related to the washout of enzymes into the circulation as the blocked vessel is opened.

A client in the outpatient clinic has a new diagnosis of peripheral artery disease (PAD). Which of the following medication categories should the nurse plan to include when providing client teaching about PAD management? a. Statins b. Vitamins c. Thrombolytics d. Anticoagulants

ANS: A Current research indicates that statin use by clients with PAD improves multiple outcomes. There is no research that supports the use of the other medication categories in PAD.

The health care provider prescribes an infusion of argatroban and daily partial thromboplastin time (PTT) testing for a client with venous thrombo-embolism (VTE). Which of the following actions should the nurse include in the plan of care? a. Avoid giving any IM medications to prevent localized bleeding. b. Discontinue the infusion for PTT values greater than 50 seconds. c. Monitor posterior tibial and dorsalis pedis pulses with the Doppler. d. Have vitamin K available in case reversal of the argatroban is needed.

ANS: A IM injections are avoided in clients receiving anticoagulation. A PTT of 50 seconds is within the therapeutic range. Vitamin K is used to reverse warfarin. Pulse quality is not affected by VTE.

Which assessment finding in a client admitted with chronic heart failure requires the most rapid action by the nurse? a. Oxygen saturation of 88% b. Weight gain of 1 kg c. Apical pulse rate of 106 beats/minute d. Urine output of 50 mL over 2 hours

ANS: A In a person with HF, oxygen saturation of the blood may be reduced because the blood is not adequately oxygenated in the lungs. Administration of oxygen, if the O2 saturation is less than 90%, can improve tissue oxygenation. Thus, appropriate use of oxygen therapy helps relieve dyspnea and fatigue. An increase in apical pulse rate, 1-kg weight gain, and decreases in urine output also indicate worsening heart failure and require rapid nursing actions, but the low oxygen saturation rate requires the most immediate nursing action.

Which of the following actions is priority when caring for a client admitted with acute decompensated heart failure (ADHF) who is receiving a nitrate? a. Monitor blood pressure frequently. b. Encourage client to ambulate in room. c. Titrate nitrate rate slowly before discontinuing. d. Teach client about safe home use of the medication.

ANS: A Nitrates cause vasodilation therefore BP should be frequently monitored. Since the client is likely to have orthostatic hypotension, the client should not be encouraged to ambulate. Nitrate does not require titration and the priority is not to teach about safe use at home.

6. The nurse is caring for a client with hypertension and has a prescription for nadolol. Which of the following assessment findings should the nurse report to the health care provider before administering this medication? a. Asthma b. Peptic ulcer disease c. Alcohol dependency d. Myocardial infarction

ANS: A Nonselective b-adrenergic blockers can cause bronchospasm, putting the client with a history of asthma at high risk. b-adrenergic blockers will have no effect on the client's peptic ulcer disease or alcohol dependency. b-adrenergic blocker therapy is recommended after MI.

The nurse is caring for a client with heart failure with reduced ejection fraction. Which of the following laboratory values should the nurse expect to assess in the client related to ejection fraction? a. 40% b. 60% c. 80% d. 90%

ANS: A Normal EF is greater than 55% of the ventricular volume. Patients with HF-REF requiring specialist intervention generally have an EF less than or equal to 40%.

The nurse is admitting a client to the emergency department with a history of an abdominal aortic aneurysm with severe back pain and absent pedal pulses. Which of the following actions should the nurse take first? a. Obtain the blood pressure. b. Ask the client about tobacco use. c. Draw blood for ordered laboratory testing. d. Assess for the presence of an abdominal bruit.

ANS: A Since the client appears to be experiencing aortic dissection, the nurse's first action should be to determine the hemodynamic status by assessing blood pressure. The other actions also may be done, but they will not provide information that will determine what interventions are needed immediately for this client.

Four days after having a myocardial infarction (MI), a client who is scheduled for discharge asks for assistance with all the daily activities, saying, "I don't understand how to care for myself." Based on this information, which of the following nursing diagnoses is appropriate? a. Ineffective health management related to insufficient knowledge b. Activity intolerance related to physical deconditioning c. Ineffective denial related to ineffective coping strategies d. Social isolation related to insufficient personal resources

ANS: A The client data indicate ineffective health management related to lack of knowledge of disease process, and care after discharge. The other nursing diagnoses may be appropriate for some clients after an MI, but the data for this client do not support denial, activity intolerance, or social isolation.

The nurse is caring for a client who had a stroke resulting from a ruptured aneurysm and subarachnoid hemorrhage. Which of the following interventions should be included in the care plan? a. Applying compression gradient stockings b. Assisting to dangle on edge of bed and assess for dizziness c. Encouraging client to cough and deep breathe every 4 hours d. Inserting an oropharyngeal airway to prevent airway obstruction

ANS: A The client with a subarachnoid hemorrhage usually has minimal activity to prevent cerebral vasospasm or further bleeding and is at risk for venous thrombo-embolism (VTE). Activities such as coughing and sitting up that might increase intracranial pressure (ICP) or decrease cerebral blood flow are avoided. Because there is no indication that the client is unconscious, an oropharyngeal airway is inappropriate.

4. The nurse is reviewing the laboratory test results for a client who has recently been diagnosed with hypertension. Which result is most important to communicate to the health care provider? a. Serum creatinine of 230 mcmol/L b. Serum potassium of 3.8 mmol/L c. Serum hemoglobin of 147 g/L d. Blood glucose level of 5.3 mmol/L

ANS: A The elevated creatinine indicates renal damage caused by the hypertension. The other laboratory results are normal.

Which of the following nursing actions should the nurse take first in order to assist a client with newly diagnosed stage 1 hypertension in making needed dietary changes? a. Have the client record dietary intake for 3 days. b. Give the client a detailed list of low-sodium foods. c. Teach the client about foods that are high in sodium. d. Help the client make an appointment with a dietitian.

ANS: A The initial nursing action should be assessment of the client's baseline dietary intake through a 3-day diet history. The other actions may be appropriate, but assessment of the client's baseline should occur first.

8. The nurse is admitting a client with left-sided hemiparesis who has arrived by ambulance to the emergency department. Which of the following actions should the nurse take first? a. Check the respiratory rate. b. Monitor the blood pressure. c. Send the client for a CT scan. d. Obtain the Glasgow Coma Scale score.

ANS: A The initial nursing action should be to assess the airway and take any needed actions to ensure a patent airway. The other activities should take place quickly after the CABs (circulation, airway, breathing) are completed.

A client at the clinic says, "I have always taken an evening walk, but lately my leg cramps and hurts after just a few minutes of walking. The pain goes away after I stop walking, though." Which of the following actions should the nurse implement? a. Attempt to palpate the dorsalis pedis and posterior tibial pulses. b. Check for the presence of tortuous veins bilaterally on the legs. c. Ask about any skin colour changes that occur in response to cold. d. Assess for unilateral swelling, redness, and tenderness of either leg.

ANS: A The nurse should assess for other clinical manifestations of peripheral arterial disease in a client who describes intermittent claudication. Changes in skin colour that occur in response to cold are consistent with Raynaud's phenomenon. Tortuous veins on the legs suggest venous insufficiency. Unilateral leg swelling, redness, and tenderness point to venous thrombo-embolism (VTE).

The nurse is admitting a client to the emergency department with severe chest pain and gives the following list of medications taken at home to the nurse. Which of the following medications has the most immediate implications for the client's care? a. Sildenafil b. Furosemide c. Diazepam d. Captopril

ANS: A The nurse will need to avoid giving nitrates to the client because nitrate administration is contraindicated in clients who are using sildenafil because of the risk of sudden death caused by vasodilation. The other home medications also should be documented and reported to the health care provider but do not have as immediate an impact on decisions about the client's treatment.

The nurse is admitting a client who has chest pain is to the emergency department and all the following diagnostic tests are prescribed. Which of the following tests should the nurse arrange to be completed first? a. Electrocardiogram (ECG) b. Computed tomography (CT) scan c. Chest x-ray d. Troponin level

ANS: A The priority for the client is to determine whether an acute myocardial infarction (AMI) is occurring so that reperfusion therapy can begin as quickly as possible. ECG changes occur very rapidly after coronary artery occlusion. Troponin levels will increase after about 3 hours. Data from the CT scan and chest x-ray may impact the client's care but are not helpful in determining whether the client is experiencing a myocardial infarction (MI).

The nurse is evaluating the outcomes of preoperative teaching with a client scheduled for a coronary artery bypass graft (CABG) using the internal mammary artery. Which of the following client statements indicates that additional teaching is needed? a. "I will have incisions in my leg where they will remove the vein." b. "They will circulate my blood with a machine during the surgery." c. "I will need to take an Aspirin a day after the surgery to keep the graft open." d. "They will use an artery near my heart to bypass the area that is obstructed."

ANS: A When the internal mammary artery is used there is no need to have a saphenous vein removed from the leg. The other statements by the client are accurate and indicate that the teaching has been effective.

The nurse is receiving a change-of-shift report. Which of the following clients should the nurse see first? a. A client with right-sided weakness who has an infusion of tPA prescribed b. A client who has atrial fibrillation and a new prescription for warfarin c. A client who experienced a transient ischemic attack yesterday who has a dose of Aspirin due Aspirin due d. A client with a subarachnoid hemorrhage 2 days ago who has nimodipine scheduled

ANS: A tPA needs to be infused within the first few hours after stroke symptoms start in order to be effective in minimizing brain injury. The other medications also should be given as quickly as possible, but timing of the medications is not as critical.

2. Which of the following BP findings by the nurse indicates that no changes in therapy are needed for a client with stage 1 hypertension who has a history of heart failure? a. 108/64 mm Hg b. 128/76 mm Hg c. 140/90 mm Hg d. 136/82 mm Hg

ANS: B The goal for antihypertensive therapy for a client with hypertension and heart failure is a BP of <130/80 mm Hg. The BP of 108/64 may indicate overtreatment of the hypertension and an increased risk for adverse effects of drugs. The other two blood pressures indicate a need for modifications in the client's treatment.

Which of the following approaches to preventing a recurrence of sudden cardiac death is the most common? a. Long-term Aspirin therapy b. Implantable cardioverter-defibrillator c. Administration of amiodarone d. Continuous Holter monitoring

ANS: B The most common approach to preventing a recurrence is the use of an implantable cardioverter-defibrillator (ICD). Research has shown survival rates are better with an ICD than with drug therapy alone. Drug therapy with amiodarone may be used in conjunction with an ICD to decrease episodes of ventricular dysrhythmias. Continuous monitoring will not prevent a recurrence. Aspirin will not prevent a recurrence of SCD.

The nurse is assessing a client with possible peripheral artery disease (PAD) and obtains a brachial BP of 140/80 and an ankle pressure of 110/70. Which of the following values is the client's ankle-brachial index (ABI)? a. 0.66 b. 0.79 c. 60 d. 406

ANS: B 0.78 or 0.79. The ABI is calculated by dividing the ankle systolic BP by the brachial systolic BP.

The nurse is conducting a health history on a client with heart failure. Which of the following conditions in the client's health history is a precipitating cause of heart failure? a. Hyperthyroidism b. Anemia c. Hypovolemia d. Diabetes

ANS: B Anemia is a precipitating cause of heart failure. Also, hypovolemia and hypothyroidism are precipitating causes. Diabetes is not a precipitating cause of heart failure.

The nurse is admitting a client with a myocardial infarction (MI) to the intensive care unit. Which of the following actions should the nurse carry out first? a. Obtain the blood pressure. b. Attach the cardiac monitor. c. Assess the peripheral pulses. d. Auscultate the breath sounds.

ANS: B Because dysrhythmias are the most common complication of MI, the first action should be to place the client on a cardiac monitor. The other actions also are important and should be accomplished as quickly as possible

. Which of the following nursing actions should be included in the plan of care for a client who has had endovascular repair of an abdominal aortic aneurysm? a. Record hourly chest tube drainage. b. Monitor fluid intake and urine output. c. Check the abdominal wound for redness or swelling. d. Teach the reason for a prolonged rehabilitation process.

ANS: B Because renal artery occlusion can occur after endovascular repair, the nurse should monitor parameters of renal function such as intake and output. Chest tubes will not be needed for endovascular surgery, the recovery period will be short, and there will not be an abdominal wound.

The nurse is providing teaching to a client with critical limb ischemia. Which of the following client statements indicate further teaching is required? a. "I will have to buy some loose clothing that does not bind across my legs or waist." b. "I will use a heating pad on my feet at night to increase the circulation and warmth in my feet." c. "I will walk to the point of pain, rest, and walk again until I develop pain for a half hour daily." d. "I will change my position every hour and avoid long periods of sitting with my legs down."

ANS: B Because the client has impaired circulation and sensation to the feet, the use of a heating pad could lead to burns. The other client statements are correct and indicate that teaching has been successful.

Which of the following information collected by the nurse who is admitting a client with chest pain suggests that the pain is caused by an acute myocardial infarction? a. The pain increases with deep breathing. b. The pain has persisted longer than 30 minutes. c. The pain worsens when the client raises the arms. d. The pain is relieved after the client takes nitroglycerin

ANS: B Chest pain that lasts for 20 minutes or more is characteristic of an acute myocardial infarction. Changes in pain that occur with raising the arms or with deep breathing are more typical of pericarditis or musculo-skeletal pain. Stable angina is usually relieved when the client takes nitroglycerin.

9. Which of the following responses by a client that is on anticoagulant therapy indicates the need for further teaching? a. "I can still have a glass of wine with my dinner." b. "For pain relief I will take ibuprofen." c. "I take my pills at two o'clock every day." d. "I will use an electric razor for shaving."

ANS: B Clients on anticoagulant therapy should avoid all NSAIDs therefore ibuprofen should not be taken for pain relief. It is acceptable to have an alcohol intake of a glass of wine daily. It is important that medications be taken at the same time every day. Clients are taught to avoid the use of a straight razor.

9. The nurse is providing teaching to a client with newly diagnosed Raynaud's phenomenon about how to manage the condition. Which of the following behaviours by the client indicate that the teaching has been effective? a. The client avoids the use of Aspirin and nonsteroidal anti-inflammatory drugs (NSAIDs). b. The client exercises indoors during the winter months. c. The client places the hands in hot water when they turn pale. d. The client takes pseudoephedrine for cold symptoms.

ANS: B Clients should avoid temperature extremes by exercising indoors when it is cold. To avoid burn injuries, the client should use warm, rather than hot, water to warm the hands. Pseudoephedrine is a vasoconstrictor and should be avoided. There is no reason to avoid taking Aspirin and NSAIDs with Raynaud's phenomenon.

1. The nurse is providing nutritional teaching to a client with hypertension. Which of the following food groups should the nurse tell the client that they should have 4-5 daily servings? a. Whole grains b. Vegetables c. Meat, fish, and poultry d. Fat-free dairy food

ANS: B Clients should be taught to have 4-5 daily servings of fruits and vegetables. Whole grains should be 7-8 servings per day. Meat, fish, and poultry are limited to less than 170 g/day. Low-fat dairy foods should be 2-3 servings per day.

The nurse identifies the nursing diagnosis of impaired verbal communication for a client with expressive aphasia. Which of the following actions should the nurse implement to help the client communicate? a. Have the client practice facial and tongue exercises. b. Ask simple questions that the client can answer with "yes" or "no."c. Develop a list of words that the client can read and practice reciting. d. Prevent embarrassing the client by changing the subject if the client does not respond.

ANS: B Communication will be facilitated and less frustrating to the client when questions that require a "yes" or "no" response are used. When the language areas of the brain are injured, the client might not be able to read or recite words, which will frustrate the client without improving communication. Expressive aphasia is caused by damage to the language areas of the brain, not by the areas that control the motor aspects of speech. The nurse should allow time for the client to respond.

The nurse is caring for a client who had a sclerotherapy for treatment of superficial varicose veins and is a service-counter worker. Which of the following information should the nurse include when providing discharge teaching to the client? a. Sitting at the work counter, rather than standing, is recommended. b. Compression stockings should be applied before getting out of bed. c. Exercises such as walking or jogging cause recurrence of varicosities. d. Taking one Aspirin daily will help prevent clotting around venous valves.

ANS: B Compression stockings are applied with the legs elevated to reduce pressure in the lower legs. Walking is recommended to prevent recurrent varicosities. Sitting and standing are both risk factors for varicose veins and venous insufficiency. An Aspirin a day is not adequate to prevent venous thrombosis and would not be recommended to the client who had just had sclerotherapy.

Several weeks after a stroke, a client has urinary incontinence resulting from an impaired awareness of bladder fullness. For an effective bladder training program, which of the following nursing interventions will be best to include in the plan of care? a. Limit fluid intake to 1 200 mL daily to reduce urine volume. b. Assist the client onto the bedside commode every 2 hours. c. Perform intermittent catheterization after each voiding to check for residual urine. d. Use an external "condom" catheter to protect the skin and prevent embarrassment.

ANS: B Developing a regular voiding schedule will prevent incontinence and may increase client awareness of a full bladder. A 1 200 mL fluid restriction may lead to dehydration. Intermittent catheterization and use of a condom catheter are appropriate in the acute phase of stroke but should not be considered solutions for long-term management because of the risks for urinary tract infection (UTI) and skin breakdown.

2. The nurse is obtaining a health history from a client who has a 5-cm thoracic aortic aneurysm that was discovered during a routine chest x-ray. Which of the following symptoms should the nurse expect to assess in the client? a. Back or lumbar pain b. Difficulty swallowing c. Abdominal tenderness d. Changes in bowel habits

ANS: B Difficulty swallowing may occur with a thoracic aneurysm because of pressure on the esophagus. The other symptoms will be important to assess for in clients with abdominal aortic aneurysms.

The nurse is caring for a client who had a stroke and is in the acute phase of care. Which of the following systems is priority? a. Neurological system b. Respiratory system c. Gastro-intestinal system d. Genito-urinary system

ANS: B During the acute phase following a stroke, management of the respiratory system is a nursing priority. Stroke patients are particularly vulnerable to respiratory problems as it has been shown that respiratory muscle strength decreases following stroke. Advancing age and immobility increase the risk for atelectasis and pneumonia.

The nurse is caring for a client who was admitted the previous day to the coronary care unit with an acute myocardial infarction. Which of the following information should the nurse include in the teaching plan for the client? a. Typical emotional responses to AMI b. When client cardiac rehabilitation will begin. c. Discharge drugs such as Aspirin and b-blockers. d. The pathophysiology of coronary artery disease.

ANS: B Early after an AMI, the client will want to know when resumption of usual activities can be expected. At this time, the client's anxiety level or denial will prevent good understanding of complex information such as coronary artery disease (CAD) pathophysiology. Teaching about discharge medications should be done when the time for discharge is closer. The nurse should support the client by decreasing anxiety rather than discussing the typical emotional response to myocardial infarction (MI).

The nurse is teaching a client's family about immediate stroke care. Which of the following information should the nurse include in teaching plan? a. Hypotension post stroke is normal. b. Antihypertensive medication is administered if the mean arterial pressure is >130 mm Hg. c. Diuretic ordered in the systolic BP is >160 mm Hg. d. Withholding medications until the degree of dysphasia is known.

ANS: B Elevated BP is common immediately after a stroke and may be a protective response to maintain cerebral perfusion. Immediately following ischemic stroke, use of drugs to lower BP is recommended only if BP is markedly increased (mean arterial pressure >130 mm Hg or systolic BP >220 mm Hg). Withholding medications can be dangerous; medications do not have to be given by the oral route

A client with ST segment elevation in several electrocardiographic (ECG) leads is admitted to the emergency department (ED) and diagnosed as having an ST-segment-elevation myocardial infarction (STEMI). Which of the following questions should the nurse ask to determine whether the client is a candidate for fibrinolytic therapy? a. "Do you take Aspirin on a daily basis?" b. "What time did your chest pain begin?" c. "Is there any family history of heart disease?" d. "Can you describe the quality of your chest pain?"

ANS: B Fibrinolytic therapy should be started within 6 hours of the onset of the myocardial infarction (MI), so the time at which the chest pain started is a major determinant of the appropriateness of this treatment. The other information also will be needed, but it will not be a factor in the decision about fibrinolytic therapy.

A client is admitted to the hospital with dysphasia and right-sided weakness that resolves in a few hours. The nurse will anticipate teaching the client about a. Alteplase (tPA). b. Aspirin. c. Warfarin. d. Nimodipine.

ANS: B Following a TIA, clients typically are started on medications such as Aspirin to inhibit platelet function and decrease stroke risk. tPA is used for acute ischemic stroke. Warfarin is usually used for clients with atrial fibrillation. Nimodipine is used to prevent cerebral vasospasm after a subarachnoid hemorrhage.

The nurse is caring for a client with peripheral artery disease who is Aspirin intolerant. Which of the following medications should the nurse anticipate the health care provider prescribing for the client related to this intolerance? a. Pentoxifylline b. Clopidogrel c. Ramipril d. Warfarin

ANS: B For clients who are Aspirin intolerant clopidogrel (75 mg/day) is indicated. Pentoxifylline is used to treat intermittent claudication. Ramipril is an ACE inhibitor. Warfarin is an anticoagulant and is not recommended for the prevention of coronary artery disease in clients with PAD.

The nurse is caring for a client who has survived a sudden cardiac death (SCD) event and has no evidence of an acute myocardial infarction. Which of the following information should the nurse teach the client? a. That sudden cardiac death events rarely reoccur b. About the purpose of outpatient Holter monitoring c. How to self-administer low-molecular-weight heparin d. To limit activities after discharge to prevent future events

ANS: B Holter monitoring is used to determine whether the client is experiencing dysrhythmias such as ventricular tachycardia during normal daily activities. SCD is likely to recur. Heparin will not have any effect on the incidence of SCD, and SCD can occur even when the client is resting.

Which of the following assessment findings for a client who is receiving furosemide to treat stage 2 hypertension is most important to report to the health care provider? a. Blood glucose level of 10 mmol/L b. Blood potassium level of 3.0 mmol/L c. Early morning BP reading of 164/96 mm Hg d. Orthostatic systolic BP decrease of 12 mm Hg

ANS: B Hypokalemia is a frequent adverse effect of the loop diuretics and can cause life-threatening dysrhythmias. The health care provider should be notified of the potassium level immediately and administration of potassium supplements initiated. The elevated blood glucose and BP also indicate a need for collaborative interventions but will not require action as urgently as the hypokalemia. An orthostatic drop of 12 mm Hg is common and will require intervention only if the client is symptomatic.

The nurse is caring for a client with Class III status (NYHA) heart failure and type 2 diabetes and the client asks the nurse whether heart transplant is a possible therapy. Which of the following responses by the nurse is best? a. "Since you have diabetes, you would not be a candidate for a heart transplant." b. "The choice of a client for a heart transplant depends on many different factors." c. "Your heart failure has not reached the stage in which heart transplants are

ANS: B Indications for a heart transplant include end-stage heart failure, but other factors such as coping skills, family support, and client motivation to follow the rigorous post-transplant regimen are also considered. Clients with diabetes who have well-controlled blood glucose levels may be candidates for heart transplant. Although heart transplants can be associated with many complications, this response does not address the client's question.

4. The nurse is caring for a client on the first postoperative day after an abdominal aortic aneurysm repair. Which of the following assessment findings is most important to communicate to the health care provider? a. Absence of flatus b. Loose, bloody stools c. Hypotonic bowel sounds d. Abdominal pain with palpation

ANS: B Loose, bloody stools at this time may indicate intestinal ischemia or infarction and should be reported immediately because the client may need an emergency bowel resection. The other findings are normal on the first postoperative day after abdominal surgery. DIF: Cogniti

9. The nurse is caring for a client in the intensive care unit with acute decompensated heart failure (ADHF) who has symptoms of severe dyspnea and is anxious, tachypneic, and tachycardic. All these medications have been prescribed for the client. Which of the following actions should the nurse implement first? a. Give IV diazepam 2.5 mg. b. Administer IV morphine sulphate 2 mg. c. Increase nitroglycerin infusion by 5 mcg/min. d. Increase dopamine infusion by 2 mcg/kg/min.

ANS: B Morphine improves alveolar gas exchange, improves cardiac output by reducing ventricular preload and afterload, decreases anxiety, and assists in reducing the subjective feeling of dyspnea. Diazepam may decrease client anxiety, but it will not improve the cardiac output or gas exchange. Increasing the dopamine may improve cardiac output, but it also will increase the heart rate and myocardial oxygen consumption. Nitroglycerin will improve cardiac output and may be appropriate for this client, but it will not directly reduce anxiety and will not act as quickly as morphine to decrease dyspnea.

The nurse is caring for a client with a right calf venous thrombo-embolism. Which of the following information requires immediate action by the nurse? a. Complaint of left calf pain b. New onset shortness of breath c. Red skin colour of left lower leg d. Temperature of 38°C (100.4°F)

ANS: B New onset dyspnea suggests a pulmonary embolus, which will require rapid actions such as oxygen administration and notification of the health care provider. The other findings are typical of VTE.

The health care provider prescribes clopidogrel for a client with cerebral atherosclerosis. Which of the following information should the nurse include when teaching the client about the new medication? a. Monitor and record the blood pressure daily. b. Call the health care provider if stools are tarry. c. It will dissolve clots in the cerebral arteries. d. It will reduce cerebral artery plaque formation.

ANS: B Plavix inhibits platelet function as it increases the risk for bleeding, so clients should be advised to notify the health care provider about any signs of bleeding. The medication does not lower blood pressure, decrease plaque formation, or dissolve clots.

9. The nurse is caring for a client with newly diagnosed Prinzmetal's (variant) angina and has a prescription for amlodipine. Which of the following information is accurate about amlodipine? a. Reduce the "fight or flight" response b. Decrease spasm of the coronary arteries c. Increase the force of myocardial contraction d. Help prevent clotting in the coronary arteries

ANS: B Prinzmetal's angina is caused by coronary artery spasm. Calcium channel blockers (e.g., amlodipine, nifedipine) are a first-line therapy for this type of angina. Platelet inhibitors, such as Aspirin, help prevent coronary artery thrombosis, and b-blockers decrease sympathetic stimulation of the heart. Medications or activities that increase myocardial contractility will increase the incidence of angina by increasing oxygen demand.

0. The nurse is caring for a client who had a stroke affecting the right hemisphere of the brain. Which of the following nursing diagnoses is appropriate based on knowledge of the effects of right brain damage? a. Impaired physical mobility related to decrease in muscle control (right hemiplegia). b. Risk for injury as evidenced by alteration in cognitive functioning c. Impaired verbal communication related to environmental barrier (impaired speech) d. Ineffective coping related to insufficient sense of control (depression and distress about disability)

ANS: B Right-sided brain damage typically causes denial of any deficits and poor impulse control, leading to risk for injury when the client attempts activities such as transferring from a bed to a chair. Right-sided brain damage causes left hemiplegia. Left-sided brain damage typically causes language deficits. Left-sided brain damage is associated with depression and distress about the disability.

The nurse is caring for a client who has had an acute myocardial infarction and the client asks the nurse about when sexual intercourse can be resumed. Which of the following responses by the nurse is best? a. "Most clients are able to enjoy intercourse without any complications." b. "Sexual activity uses about as much energy as climbing two flights of stairs." c. "The doctor will discuss sexual intercourse when your heart is strong enough." d. "Holding and cuddling are good ways to maintain intimacy after a heart attack."

ANS: B Sexual activity places about as much physical stress on the cardiovascular system as climbing two flights of stairs. The other responses do not directly address the client's question, or may not be accurate for this client.

. The nurse is developing a teaching plan for a client newly diagnosed with peripheral artery disease (PAD). Which of the following information should the nurse include? a. "Exercise only if you do not experience any pain." b. "It is very important that you stop smoking cigarettes." c. "Try to keep your legs elevated whenever you are sitting." d. "Put on support hose early in the day before swelling occurs."

ANS: B Smoking cessation is essential for slowing the progression of PAD to critical limb ischemia and reducing the risk of myocardial infarction and death. Circulation to the legs will decrease if the legs are elevated. Clients with PAD are taught to exercise to the point of feeling pain, rest, and then resume walking. Support hose are not used for clients with PAD.

After receiving change-of-shift report, which of the following clients admitted with heart failure should the nurse assess first? a. A client who is receiving IV nitroprusside and has a blood pressure (BP) of 100/56 b. A client who is cool and clammy, with new-onset confusion and restlessness c. A client who had dizziness after receiving the first dose of captopril d. A client who has crackles in both posterior lung bases and is receiving oxygen

ANS: B The client who has "wet-cold" clinical manifestations of heart failure is perfusing inadequately and needs rapid assessment and changes in management. The other clients also should be assessed as quickly as possible, but do not have indications of severe decreases in tissue perfusion

The nurse is caring for an older-adult client with heart failure and learns that the client lives alone and sometimes confuses the "water pill" with the "heart pill." When planning for the client's discharge the nurse will facilitate which of the following actions? a. Transfer to a dementia care service b. Referral to a home health care agency c. Placement in a long-term care facility d. Arrangements for around-the-clock care

ANS: B The data about the client suggest that assistance in developing a system for taking medications correctly at home is needed. A home health nurse will assess the client's home situation and help the client develop a method for taking the two medications as directed. There is no evidence that the client requires services such as dementia care, long-term care, or around-the-clock home care.

The nurse obtains this information from a client with prehypertension. Which of the following findings is most important to address with the client? a. Low dietary fibre intake b. No regular aerobic exercise c. BMI of 23 kg/m2 d. Drinks wine with dinner once a week

ANS: B The recommendations for preventing hypertension include exercising aerobically for 30- 60 minutes 4-7 days a week. A BMI of 23 kg/m2 is within the normal BMI range. The Dietary Approaches to Stop Hypertension (DASH) diet is high in fibre, but increasing fibre alone will not prevent hypertension from developing. The client's alcohol intake will not increase the hypertension risk.

. The nurse is caring for a client who is 3 days post myocardial infarction and the client states, "I just had a little chest pain. As soon as I get out of here, I'm going for my vacation as planned." Which of the following responses should the nurse make? a. "Where are you planning to go for your vacation?" b. "What do you think caused your chest pain episode?" c. "Sometimes plans need to change after a heart attack." d. "Recovery from a heart attack takes at least a few weeks."

ANS: B When the client is experiencing denial, the nurse should assist the client in testing reality until the client has progressed beyond this step of the emotional adjustment to MI. Asking the client about vacation plans reinforces the client's plan, which is not appropriate in the immediate post-MI period. Reminding the client in denial about the MI is likely to make the client angry and lead to distrust of the nursing staff.

A client with a venous thrombo-embolism (VTE) is started on enoxaparin and warfarin. The client asks the nurse why two medications are necessary. Which of the following responses by the nurse is accurate? a. "Administration of two anticoagulants reduces the risk for recurrent venous thrombosis." b. "Enoxaparin will start to dissolve the clot, and warfarin will prevent any more clots from occurring." c. "The enoxaparin will work immediately, but the warfarin takes several days to have an effect on coagulation." d. "Because of the potential for a pulmonary embolism, it is important for you to have more than one anticoagulant."

ANS: C Low-molecular-weight heparin (LMWH) is used because of the immediate effect on coagulation and discontinued once the international normalized ratio (INR) value indicates that the warfarin has reached a therapeutic level. LMWH has no thrombolytic properties. The use of two anticoagulants is not related to the risk for pulmonary embolism, and two are not necessary to reduce the risk for another VTE.

The home health nurse is visiting a client with chronic heart failure who has prescriptions for a diuretic, an ACE-inhibitor, and a low-sodium diet and tells the nurse about a 2.3 kg weight gain in the last 3 days. Which of the following actions should the nurse do first? a. Ask the client to recall the dietary intake for the last 3 days. b. Question the client about the use of the prescribed medications. c. Assess the client for clinical manifestations of acute heart failure. d. Teach the client about the importance of dietary sodium restrictions.

ANS: C The development of dependent edema or a sudden weight gain of more than 2 kg in 2 days is often indicative of exacerbated HF. It is important that the client be immediately assessed for other clinical manifestations of decompensation, such as lung crackles. A dietary recall to detect hidden sodium in the diet, reinforcement of sodium restrictions, and assessment of medication compliance may be appropriate interventions but are not the first nursing actions indicated.

Following an acute myocardial infarction, a client ambulates in the hospital hallway. When the nurse is evaluating the client's response, which of the following assessment data would indicate that the exercise level should be decreased? a. BP changes from 118/60 to 126/68 mm Hg. b. Oxygen saturation drops from 100% to 98%. c. Heart rate increases from 66 to 90 beats/minute. d. Respiratory rate goes from 14 to 22 breaths/minute.

ANS: C A change in heart rate of more than 20 beats or more indicates that the client should stop and rest. The increases in BP and respiratory rate, and the slight decrease in oxygen saturation, are normal responses to exercise.

6. Which of the following statements made by a client with coronary artery disease after the nurse has completed teaching about nutritional therapy for CAD indicates that further teaching is needed? a. "I will switch from whole milk to 1% or nonfat milk."b. "I like fresh salmon and I will plan to eat it more often." c. "I will miss being able to eat peanut butter sandwiches." d. "I can have a cup of coffee with breakfast if I want one."

ANS: C Although only 30% of the daily calories should come from fats, most of the fat should come from monosaturated fats such as are found in nuts, olive oil, and canola oil. The client can include peanut butter sandwiches as part of their diet. The other client comments indicate a good understanding of diet.

A client who has a history of a transient ischemic attack (TIA) has an order for Aspirin 160 mg daily. When the nurse is administering the medications, the client says, "I don't need the Aspirin today. I don't have any aches or pains." Which of the following actions should the nurse take? a. Document that the Aspirin was refused by the client. b. Tell the client that the Aspirin is used to prevent aches. c. Explain that the Aspirin is ordered to decrease stroke risk. d. Call the health care provider to clarify the medication order.

ANS: C Aspirin is ordered to prevent stroke in clients who have experienced TIAs. Documentation of the client's refusal to take the medication is an inadequate response by the nurse. There is no need to clarify the order with the health care provider. The Aspirin is not ordered to prevent aches and pains.

Which of the following diagnostic tests will be most useful to the nurse in determining whether a client admitted with acute shortness of breath has heart failure? a. Serum creatine kinase (CK) b. Arterial blood gases (ABGs) c. B-type natriuretic peptide (BNP) d. 12-lead electrocardiogram (ECG)

ANS: C BNP is secreted when ventricular pressures increase, as with heart failure, and elevated BNP indicates a probable or very probable diagnosis of heart failure. 12-lead ECGs, ABGs, and CK also may be used in determining the causes or effects of heart failure but are not as clearly diagnostic of heart failure as BNP.

The nurse identifies the nursing diagnosis of imbalanced nutrition: less than body requirements related to insufficient dietary intake (secondary to impaired self-feeding ability) for a client with right-sided hemiplegia. Which of the following interventions should be included in the plan of care? a. Provide a wide variety of food choices. b. Provide oral care before and after meals. c. Assist the client to eat with the left hand. d. Teach the client the "chin-tuck" technique.

ANS: C Because the nursing diagnosis indicates that the client's imbalanced nutrition is related to the right-sided hemiplegia, the appropriate interventions will focus on teaching the client to use the left hand for self-feeding. The other interventions are appropriate for clients with other etiologies for the imbalanced nutrition

The nurse is assessing a client who has chronic peripheral artery disease (PAD) of the legs and an ulcer on the left great toe. Which of the following findings should the nurse expect? a. A positive Homans' sign b. Swollen, dry, scaly ankles c. Prolonged capillary refill in all the toes d. A large amount of drainage from the ulcer

ANS: C Capillary refill is prolonged in PAD because of the slower and decreased blood flow to the periphery. The other listed clinical manifestations are consistent with chronic venous disease.

The nurse is admitting a client with left-sided homonymous hemianopsia resulting from a stroke. Which of the following interventions should the nurse include in the plan of care during the acute period of the stroke? a. Apply an eye patch to the left eye. b. Approach the client from the left side. c. Place objects needed for activities of daily living on the client's right side. d. Reassure the client that the visual deficit will resolve as the stroke progresses

ANS: C During the acute period, the nurse should place objects on the client's unaffected side. Since there is a visual defect in the left half of each eye, an eye patch is not appropriate. The client should be approached from the right side. The visual deficit may not resolve, although the client can learn to compensate for the defect

Which of the following actions should the nurse include in the plan of care for a client who is receiving sodium nitroprusside to treat a hypertensive emergency? a. Organize nursing activities so that the client has undisturbed sleep for 6-8 hours at night. b. Assist the client up in the chair for meals to avoid complications associated with immobility. c. Use an automated noninvasive blood pressure machine to obtain frequent BP measurements. d. Place the client on NPO status to prevent aspiration caused by nausea and the associated vomiting

ANS: C Frequent monitoring of BP is needed when the client is receiving rapid-acting IV antihypertensive medications. This can be most easily accomplished with an automated BP machine or arterial line. The client will require frequent assessments, so allowing 6-8 hours of undisturbed sleep is not appropriate. When clients are receiving IV vasodilators, bed rest is maintained to prevent decreased cerebral perfusion and fainting. There is no indication that this client is nauseated or at risk for aspiration, so an NPO status is unnecessary.

The nurse is assessing a client who is being investigated for possible white coat hypertension. Which of the following actions should the nurse implement first? a. Schedule the client for frequent BP checks in the clinic. b. Instruct the client about the need to decrease stress levels. c. Tell the client how to self-monitor and record BPs at home. d. Teach the client about ambulatory blood pressure monitoring.

ANS: C Having the client self-monitor BPs at home will provide a reliable indication about whether the client has hypertension. Frequent BP checks in the clinic are likely to be high in a client with white coat hypertension. Ambulatory blood pressure monitoring may be used if the data from self-monitoring is unclear. Although elevated stress levels may contribute to hypertension, instructing the client about this is unlikely to reduce BP.

3. Which of the following information should the nurse include when teaching a client with newly diagnosed hypertension? a. Dietary sodium restriction will control BP for most clients. b. Most clients are able to control BP through lifestyle changes. c. Hypertension is usually asymptomatic until significant organ damage occurs. d. Annual BP checks are needed to monitor treatment effectiveness.

ANS: C Hypertension is usually asymptomatic until target organ damage has occurred. Lifestyle changes and sodium restriction are used to help manage blood pressure, but drugs are needed for most clients. BP should be checked by the health care provider every 3-6 months.

3. The nurse is caring for a client with hypertension and has just administered the initial dose of labetalol. Which of the following actions should the nurse take? a. Encourage oral fluids to prevent dry mouth or dehydration. b. Instruct the client to ask for help if heart palpitations occur. c. Ask the client to request assistance when getting out of bed. d. Teach the client that headaches may occur with this medication.

ANS: C Labetalol decreases sympathetic nervous system activity by blocking both a- and b-adrenergic receptors, leading to vasodilation and a decrease in heart rate, which can cause severe orthostatic hypotension. Heart palpitatio

. To assist the client with coronary artery disease (CAD) in making appropriate dietary changes, which of the following nursing interventions will be most effective? a. Instruct the client that a diet containing no saturated fat and minimal sodium will be necessary. b. Emphasize the increased risk for cardiac problems unless the client makes the dietary changes. c. Assist the client to modify favourite high-fat recipes by using polyunsaturated oils when possible. d. Provide the client with a list of low-sodium, low-cholesterol foods that should be included in the diet.

ANS: C Lifestyle changes are more likely to be successful when consideration is given to the client's values and preferences. The highest percentage of calories from fat should come from polyunsaturated fats. Although low-sodium and low-cholesterol foods are appropriate, providing the client with a list alone is not likely to be successful in making dietary changes. Removing saturated fat from the diet completely is not a realistic expectation. Telling the client about the increased risk without assisting further with strategies for dietary change is unlikely to be successful.

8. The nurse is teaching the client with heart failure about a 2 g sodium diet. Which of the following foods should the nurse explain to the client that need to be restricted? a. Canned and frozen fruits b. Fresh or frozen vegetables c. Milk, yogourt, and other milk products. d. Eggs and other high-cholesterol foods.

ANS: C Milk and yogourt naturally contain a significant amount of sodium, and intake of these should be limited for clients on a diet that limits sodium to 2 g daily. Other milk products, such as processed cheeses, have very high levels of sodium and are not appropriate for a 2 g sodium diet. The other foods listed have minimal levels of sodium and can be eaten without restriction.

A client tells the health care provider about experiencing cold, numb fingers when running during the winter and is diagnosed with Raynaud's phenomenon. Based upon this diagnosis, the client should be investigated for which of the following conditions? a. Hypertension b. Hyperlipidemia c. Autoimmune disorders d. Coronary artery disease

ANS: C Secondary Raynaud's phenomenon may occur in conjunction with autoimmune diseases such as rheumatoid arthritis, and clients should be screened for autoimmune disorders. Raynaud's phenomenon is not associated with hyperlipidemia, hypertension, or coronary artery disease.

The nurse is admitting a client with a history of hypertension and is being treated with a diuretic and an angiotensin-converting enzyme (ACE) inhibitor to the emergency department. The client has symptoms of a severe headache and has a BP of 240/118 mm Hg. Which of the following questions should the nurse ask first? a. Did you take any acetaminophen today? b. Do you have any recent stressful events in your life? c. Have you been consistently taking your medications? d. Have you recently taken any antihistamine medications?

ANS: C Sudden withdrawal of antihypertensive medications can cause rebound hypertension and hypertensive crisis. Although many over-the-counter medications can cause hypertension, antihistamines and acetaminophen do not increase BP. Stressful events will increase BP but not usually to the level seen in this client.

The nurse working in the heart failure clinic will know that teaching for a client with newly diagnosed heart failure has been effective when the client does which of the following actions? a. Uses an additional pillow to sleep when feeling short of breath at night. b. Tells the home care nurse that furosemide is taken daily at bedtime. c. Calls the clinic when the weight increases from 56 to 59 kg in 2 days. d. Says that the nitroglycerin patch will be used for any chest pain that develops.

ANS: C Teaching for a client with heart failure includes information about the need to weigh daily and notify the health care provider about an increase of more than 2 kg in a 2 day period. Nitroglycerin patches are used primarily to reduce preload (not to prevent chest pain) in clients with heart failure and should be used daily, not on an "as necessary" basis. Diuretics should be taken earlier in the day to avoid nocturia and sleep disturbance. The client should call the clinic if increased orthopnea develops, rather than just compensating by elevating the head of the bed further.

The charge nurse observes a new RPN (LPN) doing discharge teaching for a client who is hypertensive and has a new prescription for enalapril. Which of the following actions by the new RPN (LPN) should cause the charge nurse to intervene in the client's care? a. Check the BP with a home BP monitor every day b. Move slowly when moving from lying to standing. c. Increase the dietary intake of high-potassium foods. d. Make an appointment with the dietitian for teaching.

ANS: C The ACE inhibitors cause retention of potassium by the kidney, so hyperkalemia is a possible adverse effect. The other teaching by the new RPN (LPN) is appropriate for a client with newly diagnosed hypertension who has just started therapy with enalapril.

6. The nurse is administering a fibrinolytic agent to a client with an acute myocardial infarction. Which of the following assessments should cause the nurse to stop the drug infusion? a. Bleeding from the gums b. Surface bleeding from the IV site c. A decrease in level of consciousness d. A nonsustained episode of ventricular tachycardia

ANS: C The change in level of consciousness indicates that the client may be experiencing intracranial bleeding, a possible complication of fibrinolytic therapy. Bleeding of the gums and prolonged bleeding from IV sites are expected adverse effects of the therapy. The nurse should address these by avoiding any further injuries, but they are not an indication to stop infusion of the fibrinolytic medication. A nonsustained episode of ventricular tachycardia is a common reperfusion dysrhythmia and may indicate that the therapy is effective.

The nurse is caring for a client who has had a stroke and has a new prescription to attempt oral feedings. After assessing the client's gag reflex, which of the following actions should the nurse do next? a. Order a varied puréed diet. b. Assess the client's appetite. c. Assist the client into a chair. d. Offer the client a sip of juice.

ANS: C The client should be as upright as possible before attempting feeding to make swallowing easier and decrease aspiration risk. To assess swallowing ability, the nurse should initially offer water or ice to the client. Puréed diets are not recommended because the texture is too smooth. The client may have a poor appetite, but the oral feeding should be attempted regardless.

Which of the following actions should the nurse in the hypertension clinic take in order to obtain an accurate baseline blood pressure (BP) for a new client? a. Obtain a BP reading in each arm and average the results. b. Deflate the BP cuff at a rate of 5-10 mm Hg/second. c. Have the client sit in a chair. d. Assist the client to the supine position for BP measurements.

ANS: C The client should be seated to assess the initial BP and P. The BP is obtained in both arms, but the results of the two arms are not averaged. The client does not need to be in the supine position. The cuff should be deflated at 2-3 mm Hg/second.

3. An outpatient who has heart failure returns to the clinic after 2 weeks of therapy with an ACE inhibitor. Which of these assessment findings is most important for the nurse to report to the health care provider? a. Pulse rate of 56 b. 2+ pedal edema c. BP of 88/42 mm Hg d. Complaints of fatigue

ANS: C The client's BP indicates that the dose of the ACE inhibitor may need to be decreased because of hypotension. Bradycardia is a frequent adverse effect of b-adrenergic blockade, but the rate of 56 is not unusual with b-blocker therapy. b-adrenergic blockade initially will worsen symptoms of heart failure in many clients, and clients should be taught that some increase in symptoms, such as fatigue and edema, is expected during the initiation of therapy with this class of drugs.

7. Three days after a myocardial infarction (MI), the client develops chest pain that increases when taking a deep breath and is relieved by leaning forward. Which of the following actions should the nurse take next? a. Palpate the radial pulses bilaterally. b. Assess the feet for peripheral edema. c. Auscultate for a pericardial friction rub. d. Check the cardiac monitor for dysrhythmias.

ANS: C The client's symptoms are consistent with the development of pericarditis, a possible complication of MI. The other assessments listed are not consistent with the description of the client's symptoms.

The nurse is caring for a client who experiences a brief episode of tinnitus, diplopia, and dysarthria with no residual effects. Which of the following actions should the nurse anticipate as treatment for this client? a. Prophylactic clipping of cerebral aneurysms b. Heparin via continuous intravenous infusion c. Oral administration of low dose Aspirin therapy d. Therapy with tissue plasminogen activator (tPA)

ANS: C The client's symptoms are consistent with transient ischemic attack (TIA), and drugs that inhibit platelet aggregation are prescribed after a TIA to prevent stroke. Continuous heparin infusion is not routinely used after TIA or with acute ischemic stroke. The client's symptoms are not consistent with a cerebral aneurysm. tPA is used only for acute ischemic stroke, not for TIA.

The nurse is providing teaching to a client about use of sublingual nitroglycerin. Which of the following client statements indicates that the teaching has been effective? a. "I can expect indigestion as an adverse effect of nitroglycerin." b. "I can only take the nitroglycerin if I start to have chest pain." c. "I will call an ambulance if I still have pain 5 minutes after taking the nitroglycerin." d. "I will help slow down the progress of the plaque formation by taking nitroglycerin."

ANS: C The emergency medical services (EMS) system should be activated when chest pain or other symptoms are not completely relieved 5 minutes after taking one nitroglycerin. Nitroglycerin can be taken to prevent chest pain or other symptoms from developing (e.g., before intercourse). Gastric upset is not an expected adverse effect of nitroglycerin. Nitroglycerin does not impact the underlying pathophysiology of coronary artery atherosclerosis.

The nurse is caring for a client who has right-sided weakness after a stroke and is attempting to use the left hand for feeding and other activities. The client's partner insists on feeding and dressing him, telling the nurse, "I just don't like to see him struggle." Which of the following nursing diagnoses is most appropriate for the client? a. Situational low self-esteem related to pattern of helplessness b. Interrupted family processes related to shift in family roles (effects of illness of a family member) c. Disabled family coping related to differing coping styles between support person and client d. Impaired nutrition: less than body requirements related to insufficient dietary intake (hemiplegia and aphasia)

ANS: C The information supports the diagnosis of disabled family coping because the client's partner does not understand the rehabilitation program. There are no data supporting low self-esteem, and the client is attempting independence. The data do not support an interruption in family processes because this may be a typical pattern for the couple. There is no indication that the client has impaired nutrition.

During change-of-shift report, the nurse obtains this information about a client who is hypertensive and received the first dose of propranolol during the previous shift. Which of the following information indicates that the client needs immediate intervention? a. The client's most recent BP reading is 156/94 mm Hg. b. The client's pulse has dropped from 64 to 58 beats/minute. c. The client has developed wheezes throughout the lung fields. d. The client complains that the fingers and toes feel quite cold.

ANS: C The most urgent concern for this client is the wheezes, which indicate that bronchospasm (a common adverse effect of the noncardioselective b-adrenergic blockers) is occurring. The nurse should immediately obtain an oxygen saturation measurement, apply supplemental oxygen, and notify the health care provider. The mild decrease in heart rate and complaint of cold fingers and toes do not require any change in therapy. The BP reading may indicate that a change in medication type or dose may be indicated; however, this is not as urgently needed as addressing the bronchospasm

The nurse is caring for a client with right-sided heart failure who asks the nurse what caused the heart failure. Which of the following causes is the primary cause of right-sided heart failure? a. Cor pulmonale b. Chronic pulmonary hypertension c. Left-sided heart failure d. Acute decompensated heart failure

ANS: C The primary cause of right-sided failure is left-sided failure. In this situation, left-sided failure results in pulmonary congestion and increased pressure in the blood vessels of the lungs (pulmonary hypertension).

The nurse has just received a change-of-shift report about the following four clients. Which client should the nurse assess first? a. 38-year-old who has pericarditis and is complaining of sharp, stabbing chest pain b. 45-year-old who had a myocardial infarction (MI) 4 days ago and is anxious about the planned discharge c. 51-year-old with unstable angina who has just returned to the unit after having a percutaneous coronary intervention (PCI) d. 60-year-old with variant angina who is to receive a scheduled dose of nifedipine (Adalat)

ANS: C This client is at risk for bleeding from the arterial access site for the PCI, so the nurse should assess the client's blood pressure, pulse, and the access site immediately. The other clients also should be assessed as quickly as possible, but assessment of this client has the highest priority.

The nurse is caring for a client who has had severe chest pain for several hours and a diagnosis of possible acute myocardial infarction. Which of the following prescribed laboratory tests should the nurse monitor to help determine the diagnosis? a. Homocysteine b. C-reactive protein c. Cardiac-specific troponin I and troponin T d. High-density lipoprotein (HDL) cholesterol

ANS: C Troponin levels increase about 3-12 hours after the onset of myocardial infarction (MI). The other laboratory data are useful in determining the client's risk for developing coronary artery disease (CAD) but are not helpful in determining whether an acute MI is in progress.

4. The nurse receives a verbal report that a client has an occlusion of the left posterior cerebral artery. Which of the following findings should the nurse anticipate? a. Dysphasia b. Confusion c. Visual deficits d. Poor judgement

ANS: C Visual disturbances are expected with posterior cerebral artery occlusion. Aphasia occurs with middle cerebral artery involvement. Cognitive deficits and changes in judgement are more typical of anterior cerebral artery occlusion.

Which of the following clients is less likely to enroll in a cardiac rehabilitation program? a. A 64-year-old male who has diabetes b. A 51-year-old male who has a same-sex partner c. A 52-year-old single female d. A 39-year-old male with two children

ANS: C Women are 36% less likely to enroll in cardiac rehabilitation programs.

The nurse is admitting a client who began experiencing right-sided arm and leg weakness to the emergency department. In which order should the nurse implement these actions included in the stroke protocol? (Select all that apply.) a. Obtain CT scan without contrast. b. Infuse tissue plasminogen activator ( c. Administer oxygen to keep O2 saturation >95%. d. Use National Institute of Health Stroke Scale to assess client.

ANS: C, D, A, B The initial actions should be those that help with circulation, airway, and breathing. Baseline neurological assessments should be done next. A CT scan will be needed to rule out hemorrhagic stroke before tPA can be administered.

The nurse is providing teaching to a client with chronic venous insufficiency who has a venous ulcer on the right lower leg. Which of the following topics should the nurse include in the teaching plan? a. Adequate carbohydrate intake b. Prophylactic antibiotic therapy c. Application of compression to the leg d. Methods of keeping the wound area dry

ANS: CCompression of the leg is essential to healing of venous ulcers in clients with chronic venous insufficiency. High dietary intake of protein, rather than carbohydrates, is needed. Prophylactic antibiotics are not routinely used for venous ulcers. Moist environment dressings are used to hasten wound healing.

9. The nurse has just finished teaching a client who is hypertensive about the newly prescribed quinapril. Which of the following client statements indicates that more teaching is needed? a. "The medication may not work as well if I take any Aspirin." b. "My health care provider may order a blood potassium level occasionally." c. "I will call my health care provider if I notice that I have a frequent cough." d. "I won't worry if I have a little swelling around my lips and face."

ANS: D Angioneurotic edema occurring with angiotensin-converting enzyme (ACE) inhibitor therapy is an indication that the ACE inhibitor should be discontinued. The client should be taught that if any swelling of the face or oral mucosa occurs, the health care provider should be immediately notified because this could be life-threatening. The other client statements indicate that the client has an accurate understanding of ACE inhibitor therapy.

The nurse obtains all of the following information about a client in the clinic. When developing a plan to decrease stroke risk, which of the following risk factors is most important for the nurse to address? a. The client has a daily glass of wine to relax. b. The client is 13 kg above the ideal weight. c. The client works at a desk and relaxes by watching television. d. The client's blood pressure is usually about 180/90 mm Hg.

ANS: D Hypertension is the single most important modifiable risk factor and this client's hypertension is at the stage 2 level. People who drink more than 1 (for women) or 2 (for men) alcoholic beverages a day may increase risk for hypertension. Physical inactivity and obesity contribute to stroke risk but not so much as hypertension.

Which topic will the nurse plan to include in discharge teaching for a client with systolic heart failure and an ejection fraction of 38%? a. Need to participate in an aerobic exercise program several times weekly b. Use of salt substitutes to replace table salt when cooking and at the table c. Importance of making a yearly appointment with the primary care provider d. Benefits and adverse effects of angiotensin-converting enzyme (ACE) inhibitors

ANS: D The core measure for the treatment of heart failure in clients with a low ejection fraction is to receive an ACE inhibitor to decrease the progression of heart failure. Aerobic exercise may not be appropriate for a client with this level of heart failure, salt substitutes are not usually recommended because of the risk of hyperkalemia, and the client will need to see the primary care provider more frequently than annually.

7. The nurse is caring for a 52-year-old client who has no previous history of hypertension or other health problems and has suddenly developed a BP of 188/106 mm Hg. After reconfirming the BP, which of the following information is best for the nurse to tell the client? a. A BP recheck should be scheduled in a few weeks. b. The dietary sodium and fat content should be decreased. c. There is an immediate danger of a stroke and hospitalization will be required. d. More diagnostic testing may be needed to determine the cause of the hypertension.

ANS: D A sudden increase in BP in a client over age 50 or under age 20 with no previous hypertension history or risk factors indicates that the hypertension may be secondary to some other problem. The BP will need rapid treatment and ongoing monitoring. If the client has no other risk factors, a stroke in the immediate future is unlikely. There is no indication that dietary salt or fat intake has contributed to this sudden increase in BP, and reducing intake of salt and fat alone will not be adequate to reduce this BP to an acceptable level.

The nurse notes bruising and discoloration of the right leg of a client who has just arrived in the recovery unit from having vein ligation surgery. Which of the following interventions is priority? a. Place the client in the Trendelenburg position. b. Contact the health care provider. c. Elevate the bed at the knee and put pillows under the feet. d. Elevate the legs 15 degrees to limit edema.

ANS: D After vein ligation surgery, the legs should be elevated 15 degrees to limit edema. Placing the client in the Trendelenburg position will lower the head below heart level, which is not indicated for this client. Placing pillows under the calf or elevating the bed at the knee may cause blood stasis at the calf level. Bruising and discoloration are expected after vein ligation surgery so there is no need to contact the health care provider at this time.

The nurse is discussing risk factor modification for a client who has a 4-cm abdominal aortic aneurysm. The nurse should focus client teaching on which of the following risk factors? a. Male gender b. Marfan syndrome c. Abdominal trauma history d. Uncontrolled hypertension

ANS: D All of the factors contribute to the client's risk, but only the hypertension can potentially be modified to decrease the client's risk for further expansion of the aneurysm.

The nurse is assessing a client with a possible stroke and finds that the client's aphasia started 3.5 hours previously and the blood pressure is 170/92 mm Hg. Which of these prescriptions by the health care provider should the nurse question? a. Infuse normal saline at 75 mL/hour. b. Keep head of bed elevated at least 30 degrees. c. Administer tissue plasminogen activator (tPA) per protocol. d. Titrate labetalol drip to keep BP less than 140/90 mm Hg.

ANS: D Because elevated BP may be a protective response to maintain cerebral perfusion, antihypertensive therapy is recommended only if mean arterial pressure (MAP) is >130 mm Hg or systolic pressure is >220 mm Hg. Fluid intake should be 1 500-2 000 mL daily to maintain cerebral blood flow. The head of the bed should be elevated to at least 30 degrees, unless the client has symptoms of poor tissue perfusion. tPA may be administered if the client meets the other criteria for tPA use.

The nurse is developing a health teaching plan for a 60-year-old man with the following risk factors for coronary artery disease (CAD). Which of the following risk factors should the nurse focus on when teaching the client? a. Family history of coronary artery disease b. Increased risk associated with the client's gender c. High incidence of cardiovascular disease in older people d. Elevation of the client's serum low density lipoprotein (LDL) level

ANS: D Because family history, gender, and age are nonmodifiable risk factors, the nurse should focus on the client's LDL level. Decreases in LDL will help reduce the client's risk for developing CAD.

The nurse is caring for a client who is receiving IV furosemide and morphine for the treatment of acute decompensated heart failure (ADHF) with severe orthopnea. When evaluating the client response to the medications, which of the following is the best indicator that the treatment has been effective? a. Weight loss of 1 kg overnight b. Hourly urine output greater than 60 mL c. Reduction in client complaints of chest pain d. Decreased dyspnea with the head of bed at 30 degrees

ANS: D Because the client's major clinical manifestation of ADHF is orthopnea (caused by the presence of fluid in the alveoli), the best indicator that the medications are effective is a decrease in dyspnea with the head of the bed at 30 degrees. The other assessment data also may indicate that diuresis or improvement in cardiac output has occurred but are not as specific to evaluating this client's response.

1. The nurse is caring for a client with angina who has been prescribed nadolol. Which of the following parameters should the nurse assess to determine whether the drug is effective? a. Decreased blood pressure and apical pulse rate b. Fewer complaints of having cold hands and feet c. Improvement in the quality of the peripheral pulses d. The ability to do daily activities without chest discomfort

ANS: D Because the medication is ordered to improve the client's angina, effectiveness is indicated if the client is able to accomplish daily activities without chest pain. Blood pressure (BP) and apical pulse rate may decrease, but these data do not indicate that the goal of decreased angina has been met. The noncardioselective b-blockers can cause peripheral vasoconstriction, so the nurse would not expect an improvement in peripheral pulse quality or skin temperature.

Which of the following information given by a client admitted with chronic stable angina will help the nurse confirm this diagnosis? a. The client rates the pain at a level 3-5 (0-10 scale). b. The client states that the pain "wakes me up at night." c. The client says that the frequency of the pain has increased over the last few weeks. d. The client states that the pain is resolved after taking one sublingual nitroglycerin tablet.

ANS: D Chronic stable angina is typically relieved by rest or nitroglycerin administration. The level of pain is not a consistent indicator of the type of angina. Pain occurring at rest or with increased frequency is typical of unstable angina.

The nurse will suspect that the client with stable angina is experiencing an adverse effect of the prescribed metoprolol if which of the following findings are assessed? a. The client is restless and agitated. b. The blood pressure is 190/110 mm Hg. c. The client complains about feeling anxious. d. The cardiac monitor shows a heart rate of 45.

ANS: D Clients taking b-adrenergic blockers should be monitored for bradycardia. Because this category of medication inhibits the sympathetic nervous system, restlessness, agitation, hypertension, and anxiety will not be adverse effects.

The nurse is caring for a client with a left-sided brain stroke who suddenly bursts into tears when family members visit. Which of the following actions should the nurse implement? a. Use a calm voice to ask the client to stop the crying behaviour. b. Explain to the family that depression is normal following a stroke. c. Have the family members leave the client alone for a few minutes. d. Teach the family that emotional outbursts are common after strokes.

ANS: D Clients who have left-sided brain stroke are prone to emotional outbursts, which are not necessarily related to the emotional state of the client. Depression after a stroke is common, but the suddenness of the client's outburst suggests that depression is not the major cause of the behaviour. The family should stay with the client. The crying is not within the client's control and asking the client to stop will lead to embarrassment.

The nurse is caring for a client with chronic heart failure. Which of the following conditions is a cause of chronic heart disease? a. Dysrhythmias b. Pulmonary embolus c. Myocarditis d. Congenital heart disease

ANS: D Congenital heart disease is a cause of chronic heart failure. Dysrhythmias, pulmonary embolus, and myocarditis are causes of acute heart failure.

While working in the outpatient clinic, the nurse notes that the medical record states that a client has intermittent claudication. Which of the following client statements is consistent with this information? a. "When I stand too long, my feet start to swell up." b. "Sometimes I get tired when I climb a lot of stairs." c. "My fingers hurt when I go outside in cold weather." d. "My legs cramp whenever I walk more than a block."

ANS: D Cramping that is precipitated by a consistent level of exercise is descriptive of intermittent claudication. Finger pain associated with cold weather is typical of Raynaud's phenomenon. Fatigue that occurs sometimes with exercise is not typical of intermittent claudication, which is reproducible. Swelling associated with prolonged standing is typical of venous disease.

A client asks the nurse if there are any natural products that would decrease anticoagulant effects. The nurse tells the client that which of the following natural products causes a decrease in anticoagulant effects? a. Horse chestnut b. Licorice root c. Turmeric d. Green tea

ANS: D Green tea is a natural product that would decrease anticoagulant effects. Horse chestnut, licorice root, and turmeric are natural products that would increase anticoagulant effects.

The nurse is caring for a client with a non-ST-segment-elevation myocardial infarction (NSTEMI) who is receiving heparin. Which of the following information explains the purpose of the heparin? a. Platelet aggregation is enhanced by IV heparin infusion. b. Heparin will dissolve the clot that is blocking blood flow to the heart. c. Coronary artery plaque size and adherence are decreased with heparin. d. Heparin will prevent the development of new clots in the coronary arteries.

ANS: D Heparin helps prevent the conversion of fibrinogen to fibrin and decreases coronary artery thrombosis. It does not change coronary artery plaque, dissolve already formed clots, or enhance platelet aggregation.

Immediately after repair of an abdominal aortic aneurysm, a client has absent popliteal, posterior tibial, and dorsalis pedis pulses. The legs are cool and mottled. Which of the following actions should the nurse take first? a. Wrap both the legs in warm blankets. b. Notify the surgeon and anaesthesiologist. c. Document that the pulses are absent and recheck in 30 minutes. d. Review the preoperative assessment form for data about the pulses.

ANS: D Many clients with aortic aneurysms also have peripheral arterial disease, so the nurse should check the preoperative assessment to determine whether pulses were present before surgery before notifying the health care providers about the absent pulses. Because the client's symptoms may indicate graft occlusion or multiple emboli and a possible need to return to surgery, it is not appropriate to wait 30 minutes before taking action. Warm blankets will not improve the circulation to the client's legs

5. A client who has chronic heart failure tells the nurse, "I felt fine when I went to bed, but I woke up in the middle of the night feeling like I was suffocating!" Which of the following information should the nurse document related to this assessment? a. Pulsus alternans b. Two-pillow orthopnea c. Acute bilateral pleural effusion d. Paroxysmal nocturnal dyspnea

ANS: D Paroxysmal nocturnal dyspnea is caused by the reabsorption of fluid from dependent body areas when the client is sleeping and is characterized by waking up suddenly with the feeling of suffocation. Pulsus alternans is the alternation of strong and weak peripheral pulses during palpation. Orthopnea indicates that the client is unable to lie flat because of dyspnea. Pleural effusions develop over a longer time period.

The nurse is caring for a client who has had a subarachnoid hemorrhage and is being cared for in the intensive care unit. Which of the following information about vasospasms should the nurse be aware of when planning care? a. The client's blood pressure is 100/50 mm Hg. b. Endothelin will subside the vasospasm c. The cerebro-spinal fluid (CSF) report shows red blood cells (RBCs). d. Peak time for occurrence is 7-10 days post bleed.

ANS: D Peak time for vasospasm to occur is 7-10 days after the initial bleed. In addition, release of endothelin (a potent vasoconstrictor) may play a major role in the induction of cerebral vasospasm after SAH rather than helping to relieve it. The BP is within normal limits. RBCs in the CSF are a typical clinical manifestation of a subarachnoid hemorrhage.

The nurse is caring for a client with a stroke who has progressive development of neurological deficits with increasing weakness and decreased level of consciousness (LOC). Which of the following nursing diagnoses has the highest priority for the client? a. Impaired physical mobility related to decrease in muscle strength b. Risk for injury as evidenced by alteration in cognitive functionc. Risk for impaired skin integrity as evidenced by pressure over bony prominence (immobilty) d. Risk for aspiration as evidenced by impaired ability to swallow

ANS: D Protection of the airway is the priority of nursing care for a client having an acute stroke. The other diagnoses also are appropriate, but interventions to prevent aspiration are the priority at this time.

The nurse is admitting a client with right-sided weakness that started 90 minutes earlier to the emergency department and all these diagnostic tests are prescribed. Which of the following tests should be done first? a. Electrocardiogram (ECG) b. Complete blood count (CBC) c. Chest radiograph (chest x-ray) d. Noncontrast computed tomography (CT) scan

ANS: D Rapid screening with a noncontrast CT scan is needed before administration of tissue plasminogen activator (tPA), which must be given within 4.5 hours of the onset of clinical manifestations of the stroke. The sooner the tPA is given, the smaller the area of brain injury. The other diagnostic tests give information about possible causes of the stroke and do not need to be completed as urgently as the CT scan.

The nurse is admitting a client who had a stroke and is experiencing right-sided arm and leg paralysis and facial drooping on the right side. Which of the following clinical manifestations should the nurse expect to find? a. Impulsive behaviour b. Right-sided neglect c. Hyperactive left-sided reflexes d. Difficulty in understanding commands

ANS: D Right-sided paralysis indicates a left-brain stroke, which will lead to difficulty with comprehension and use of language. The left-side reflexes are likely to be intact. Impulsive behaviour and neglect are more likely with a right-side stroke.

When a client with hypertension who has a new prescription for atenolol returns to the health clinic after 2 weeks for a follow-up visit, the BP is unchanged from the previous visit. Which of the following actions should the nurse take first? a. Provide information about the use of multiple drugs to treat hypertension. b. Teach the client about the reasons for a possible change in drug therapy. c. Remind the client that lifestyle changes also are important in BP control. d. Ask the client about whether the medication is actually being taken.

ANS: D Since nonadherence with antihypertensive therapy is common, the nurse's initial action should be to determine whether the client is taking the atenolol as prescribed. The other actions also may be implemented, but these would be done after assessing client compliance with the prescribed therapy.

The nurse is teaching a client with stage 1 hypertension about diet modifications that should be implemented. Which of the following diet choices indicates that the teaching has been effective? a. The client avoids eating nuts or nut butters. b. The client restricts intake of dietary protein. c. The client has only one cup of coffee in the morning. d. The client has a glass of low-fat milk with each meal.

ANS: D The Dietary Approaches to Stop Hypertension (DASH) recommendations for prevention of hypertension include increasing the intake of calcium-rich foods. Caffeine restriction and decreased protein intake are not included in the recommendations. Nuts are high in beneficial nutrients and 4-5 servings weekly are recommended in the DASH diet.

Which of the following actions by a nurse who is administering fondaparinux to a client with venous thrombo-embolism (VTE) indicates that more education about the medication is needed? a. The nurse avoids rubbing the injection site after giving the medication. b. The nurse injects the medication into the abdominal subcutaneous tissue. c. The nurse fails to assess the partial thromboplastin time (PTT) before administration of the medication. d. The nurse ejects the air bubble in the syringe before administering the medication.

ANS: D The air bubble is not ejected before giving fondaparinux. The other actions by the nurse are appropriate

The nurse is caring for a client who has just diagnosed with hypertension and has a new prescription for captopril. Which of the following information is important to include when teaching the client? a. Check BP daily before taking the medication. b. Increase fluid intake if dryness of the mouth is a problem. c. Include high-potassium foods such as bananas in the diet. d. Change position slowly to help prevent dizziness and falls.

ANS: D The angiotensin-converting enzyme (ACE) inhibitors frequently cause orthostatic hypotension, and clients should be taught to change position slowly to allow the vascular system time to compensate for the position change. Increasing fluid intake may counteract the effect of the medication, and the client is taught to use gum or hard candy to relieve dry mouth. The BP does not need to be checked at home by the client before taking the medication. Because ACE inhibitors cause potassium retention, increased intake of high-potassium foods is inappropriate.

The nurse is caring for a client with hyperlipidemia who has a new prescription for colestipol. Which of the following nursing actions is best when giving the medication? a. Administer the medication at the client's bedtime. b. Have the client take this medication with an Aspirin. c. Encourage the client to take the colestipol with a sip of water. d. Give the client's other medications 2 hours after the colestipol.

ANS: D The bile acid sequestrants interfere with the absorption of other drugs, and giving other medications at the same time should be avoided. Taking an Aspirin concurrently with the colestipol may increase the incidence of gastrointestinal adverse effects such as heartburn. An increased fluid intake is encouraged for clients taking the bile acid sequestrants to reduce the risk for constipation. For maximum effect, colestipol should be administered with meals.

. Which of the following electrocardiographic (ECG) change is most important for the nurse to communicate to the health care provider when caring for a client with chest pain? a. Frequent premature atrial contractions (PACs) b. Inverted P wave c. Sinus tachycardia d. ST segment elevation

ANS: D The client is likely to be experiencing an ST-segment-elevation myocardial infarction (STEMI) and immediate therapy with percutaneous coronary intervention (PCI) or fibrinolytic medications is indicated to minimize the amount of myocardial damage. The other ECG changes also may suggest a need for therapy, but not as rapidly

8. The nurse is providing teaching to a client with chronic stable angina about how to use the prescribed short-acting and long-acting nitrates. Which of the following client statements indicates that the teaching has been effective? a. "I will put on the nitroglycerin patch as soon as I develop any chest pain." b. "I will check the pulse rate in my wrist just before I take any nitroglycerin." c. "I will be sure to remove the nitroglycerin patch before using any sublingual nitroglycerin." d. "I will stop what I am doing and sit down before I put the nitroglycerin under my tongue."

ANS: D The client should sit down before taking the nitroglycerin to decrease cardiac workload and prevent orthostatic hypotension. Transdermal nitrates are used prophylactically rather than to treat acute pain and can be used concurrently with sublingual nitroglycerin. Although the nurse should check blood pressure before giving nitroglycerin, clients do not need to check the pulse rate before taking nitrates.

The nurse is caring for a client with acute coronary syndrome who has returned to the coronary care unit after having percutaneous coronary intervention and the nurse obtains these assessment data. Which of the following data indicate the need for immediate intervention by the nurse? a. Pedal pulses 1+ b. Heart rate 100 beats/minute c. Blood pressure 104/56 mm Hg d. Chest pain level 8 on a 10-point scale

ANS: D The client's chest pain indicates that restenosis of the coronary artery may be occurring and requires immediate actions, such as administration of oxygen and nitroglycerin, by the nurse. The other information indicates a need for ongoing assessments by the nurse.

The nurse is caring for a client with chronic atrial fibrillation who develops sudden severe pain, pulselessness, pallor, and coolness in the left leg. Which of the following actions should the nurse implement first? a. Elevate the left leg on a pillow. b. Apply an elastic wrap to the leg. c. Assist the client in gently exercising the leg. d. Notify the health care provider

ANS: D The client's history and clinical manifestations are consistent with acute arterial occlusion. Clinical manifestations of acute arterial ischemia include the "six Ps": pain, pallor, paralysis, pulselessness, paresthesia, and poikilothermia (adaptation of the limb to the environmental temperature, most often cool). Without immediate intervention, ischemia may progress quickly to tissue necrosis and gangrene within a few hours. If the nurse detects these signs, the nurse should immediately notify the health care provider. Elevating the leg or applying an elastic wrap will further compromise blood flow to the leg. Exercise will increase oxygen demand for the tissues of the leg.

A client with a history of several transient ischemic attacks (TIAs) arrives in the emergency department with hemiparesis and dysarthria that started 2 hours previously. Which of the following procedures should the nurse anticipate? a. Surgical endarterectomy b. Transluminal angioplasty c. Intravenous heparin administration d. Tissue plasminogen activator (tPA) infusion

ANS: D The client's history and clinical manifestations suggest an acute ischemic stroke and a client who is seen within 3-4.5 hours of stroke onset is likely to receive tPA (after screening with a CT scan). Heparin administration in the emergency phase is not indicated. Emergent carotid transluminal angioplasty or endarterectomy is not indicated for the client who is having an acute ischemic stroke.

The nurse obtains the following data when caring for a client who experienced an acute myocardial infarction 2 days previously. Which of the following information is most important to report to the health care provider? a. The client denies ever having a heart attack. b. The cardiac-specific troponin level is elevated. c. The client has occasional premature atrial contractions (PACs). d. Crackles are auscultated bilaterally in the mid-lower lobes.

ANS: D The crackles indicate that the client may be developing heart failure, a possible complication of myocardial infarction (MI). The health care provider may need to order medications such as diuretics or angiotensin-converting enzyme (ACE) inhibitors for the client. Elevation in cardiac troponin level at this time is expected. PACs are not life-threatening dysrhythmias. Denial is a common response in the immediate period after the MI.

Several hours after an open surgical repair of an abdominal aortic aneurysm, the client develops a urinary output of 20 mL/hour for 2 hours. Which of the following prescriptions should the nurse anticipate? a. An additional antibiotic b. White blood cell (WBC) count c. Decrease in IV infusion rate d. Blood urea nitrogen (BUN) level

ANS: D The decreased urine output suggests decreased renal perfusion, and monitoring of renal function is needed. There is no indication that infection is a concern, so antibiotic therapy and a WBC count are not needed. The IV rate may be increased because hypovolemia may be contributing to the client's decreased urinary output.

The nurse is administering IV nitroglycerin to a client with a myocardial infarction (MI). Which of the following actions should the nurse take to evaluate the effectiveness of the medication? a. Check blood pressure. b. Monitor apical pulse rate. c. Monitor for dysrhythmias. d. Ask about chest discomfort.

ANS: D The goal of IV nitroglycerin administration in MI is relief of chest pain by improving the balance between myocardial oxygen supply and demand. The nurse also will monitor heart rate and BP and observe for dysrhythmias, but these parameters will not indicate whether the medication is effective

The nurse is providing teaching to a client who has a prescription for transdermal nitroglycerin drug administration via reservoir. The client asks the nurse how often each day will the drug be administered. Which of the following information is the basis for the nurse's response? a. Every 4 hours while awake b. Every 6 hours around the clock c. Every 12 hours d. Once every 24 hours

ANS: D The reservoir system delivers the drug using a rate-controlled permeable membrane. The reservoir delivery system offers the advantage of steady plasma levels within the therapeutic range during 24 hours; thus, only one application a day is necessary

1. The nurse is caring for a client with sudden-onset right-sided weakness who has a CT scan and is diagnosed with an intracerebral hemorrhage. Which of the following information about the client is most important to communicate to the health care provider? a. The client's speech is difficult to understand. b. The client's blood pressure is 144/90 mm Hg. c. The client takes a diuretic because of a history of hyp

ANS: D The use of warfarin will have contributed to the intracerebral bleeding and remains a risk factor for further bleeding. Administration of vitamin K is needed to reverse the effects of the warfarin, especially if the client is to have surgery to correct the bleeding. The history of hypertension is a risk factor for the client but has no immediate effect on the client's care. The BP of 144/90 indicates the need for ongoing monitoring but not for any immediate change in therapy. Slurred speech is consistent with a left-sided stroke, and no change in therapy is indicated.

A client with a history of chronic heart failure is admitted to the emergency department (ED) with severe dyspnea and a dry, hacking cough. Which of the following actions should the nurse take first? a. Palpate the abdomen. b. Assess the orientation. c. Check the capillary refill. d. Auscultate the lung sounds.

ANS: D This client's severe dyspnea and cough indicate that acute decompensated heart failure (ADHF) is occurring. ADHF usually manifests as pulmonary edema, which should be detected and treated immediately to prevent ongoing hypoxemia and cardiac or respiratory arrest. The other assessments will provide useful data about the client's volume status and also should be accomplished rapidly, but detection (and treatment) of pulmonary complications is the


Kaugnay na mga set ng pag-aaral

History of World Civilizations to 1789

View Set

Chapter 7: Long Term Memory Encoding, Retrieval, and Consolidation

View Set

Chapter 39: Patients with Rheumatic Disorders

View Set

Science 15 EX Goes to Hell The Final Nightmare

View Set

Final Exam: Social Institutions 260 - Ott

View Set